Difference between revisions of "2004 AIME I Problems/Problem 2"
m (box fmt etc) |
|||
Line 1: | Line 1: | ||
== Problem == | == Problem == | ||
− | Set <math> A </math> consists of <math> m </math> consecutive integers whose sum is <math> 2m, </math>and set <math> B </math> consists of <math> 2m </math> consecutive integers whose sum is <math> m. </math> The absolute value of the difference between the greatest element of <math> A </math> and the greatest element of <math> B </math> is 99. Find <math> m. </math> | + | [[Set]] <math> A </math> consists of <math> m </math> consecutive integers whose sum is <math> 2m, </math>and set <math> B </math> consists of <math> 2m </math> consecutive integers whose sum is <math> m. </math> The absolute value of the difference between the greatest element of <math> A </math> and the greatest element of <math> B </math> is <math>99</math>. Find <math> m. </math> |
== Solution == | == Solution == | ||
− | Let us give the [[element]]s of our | + | Let us give the [[element]]s of our sets names: |
<math>A = \{x, x + 1, x + 2, \ldots, x + m - 1\}</math> and <math>B = \{y, y + 1, \ldots, y + 2m - 1\}</math>. So we are given that | <math>A = \{x, x + 1, x + 2, \ldots, x + m - 1\}</math> and <math>B = \{y, y + 1, \ldots, y + 2m - 1\}</math>. So we are given that | ||
+ | <cmath>2m = x + (x + 1) + \ldots + (x + m - 1) = mx + (1 + 2 + \ldots + (m - 1)) = mx + \frac{m(m -1)}2,</cmath> | ||
+ | so <math>2 = x + \frac{m - 1}2</math> and <math>x + (m - 1) = \frac{m + 3}2</math>. Also, | ||
+ | <cmath>m = y + (y + 1) + \ldots + (y + 2m - 1) = 2my + \frac{2m(2m - 1)}2,</cmath> | ||
+ | so <math>1 = 2y + (2m - 1)</math> so <math>2m = 2(y + 2m - 1)</math> and <math>m = y + 2m - 1</math>. | ||
− | + | Then by the given, <math>99 = |(x + m - 1) - (y + 2m - 1)| = \left|\frac{m + 3}2 - m\right| = \left|\frac{m - 3}2\right|</math>. <math>m</math> is a [[positive integer]] so we must have <math>99 = \frac{m - 3}2</math> and so <math>m = \boxed{201}</math>. | |
− | |||
− | |||
− | |||
− | Then by the given, <math>99 = |(x + m - 1) - (y + 2m - 1)| = |\frac{m + 3}2 - m| = |\frac{m - 3}2|</math>. <math>m</math> is a [[positive integer]] so we must have <math>99 = \frac{m - 3}2</math> and so <math>m = 201</math>. | ||
== See also == | == See also == | ||
− | + | {{AIME box|year=2004|n=I|num-b=1|num-a=3}} | |
− | |||
− | |||
− | + | [[Category:Intermediate Algebra Problems]] |
Revision as of 11:26, 27 April 2008
Problem
Set consists of consecutive integers whose sum is and set consists of consecutive integers whose sum is The absolute value of the difference between the greatest element of and the greatest element of is . Find
Solution
Let us give the elements of our sets names: and . So we are given that so and . Also, so so and .
Then by the given, . is a positive integer so we must have and so .
See also
2004 AIME I (Problems • Answer Key • Resources) | ||
Preceded by Problem 1 |
Followed by Problem 3 | |
1 • 2 • 3 • 4 • 5 • 6 • 7 • 8 • 9 • 10 • 11 • 12 • 13 • 14 • 15 | ||
All AIME Problems and Solutions |